Hello, dear friend, you can consult us at any time if you have any questions, add WeChat: daixieit

MAT00028M

MMath and MSc Examinations 2018/19

Stochastic Calculus and Black-Scholes Theory

1 (of 3).     Let (Ω , F, P) be a probability space, let (Ft )to0  be a ltration on this space

and let W be a Brownian Motion with respect to (Ft )to0 .

(a)   Define what is meant by Brownian motion.  Also, define what is meant

(c)   Show whether ξ , η defined by

5,                 if t e [0, 6) 4,          if t e [0, 3)

ξ(t)  = ,    η(t)  = 2W(W2) ,

(0,                if t > 10                          (0,         if t > 9

are random step processes. Give reasons for your answer. If any of these processes is indeed a random step process, then calculate the mean and

variance of its stochastic integral I(.). [7]

(d)   (i)   State the Itˆo Lemma in either its simple or general form.

(ii)   Use the Itˆo lemma to show that the stochastic process Z defined by Zt  = Wt2 _ t, t > 0 is a martingale.

(iii)   Use the previous part of the question to determine the quadratic variation process (W) of the Brownian motion W .  Show that the process found by you satisfies the definition of the quadratic varia- tion of a continuous square-integrable martingale. [8]

Let (Ω , F, P) be a probability space, let (Ft )to0  be a ltration on this space and let W be a Brownian Motion with respect to (Ft )to0 .

(a)   (i)   Define the spaces M2 (0, o) and Mlo(2)c (0, o).

(ii)   Show that the process A defined by At  = sin (exp (7Wt3 )) belongs to Mlo(2)c (0, o).

(iii)   Show that the Brownian motion W is not in M2 (0, o). [6]

(b)   Show that the stochastic process X defined by Xt  = sin (Wt ) satisfies the stochastic differential equation

dXt  = cos (Wt ) dWt _ Xt dt,    X0  = 0.

[4]

(c)   Let X and Y be Itˆo processes with characteristics aX , bX   and aY , bY respectively, i.e. suppose that

dXt  = aX (t) dt + bX (t) dWt

and

dYt  = aY (t) dt + bY (t) dWt .

Prove the stochastic integration by parts formula

d (XY)t  = Xt dYt + Yt dXt + bX (t) bY (t) dt.

You may assume without proof that the processes X _ Y and X + Y are

Itˆo processes. [6]

(d)   Consider the stochastic differential equation

dXt  = (Xt )γ dt + βXt dWt ,    X0  = x > 0,

where 0 < γ s 1, β > 0 are constants.  Consider the process G defined by

Gt  = exp _βWt + β 2t

and let Zt  = GtXt , t > 0. Show that Z satisfies

dZt  = (Zt )γ (Gt )1_γ dt,    Z0  = x,                           (1)

which is an ordinary differential equation (ODE). For 0 < γ < 1 solve this ODE for Zt and, hence, find the solution Xt . State the solution Xt if γ = 1 (you do not need to prove your result for the particular case γ = 1).

Hint: Apply the Itˆo formula to the Itˆo process G to determine its char- acteristics.   Next, use stochastic integration by parts for the product

GtXt  in order to derive equation (1). [9]

3 (of 3).     Suppose that σ > 0, r > 0, T > 0 and µ e R are xed.  Let (Ω , F, P) be

a probability space and let W be a Brownian motion defined on this space. Consider the Black-Scholes model, i.e.  a market consisting of a stock and a bond with prices given by stochastic processes S and B respectively. Assume that S and B are solutions to the following stochastic differential equations:

dSt  = µSt dt + σSt dWt , t > 0,

dBt  = rBt dt, t > 0.

(a)   Let (ϕ, ψ) be a trading strategy in the Black-Scholes model.

(i)   Define the wealth process associated to the trading strategy (ϕ, ψ). Furthermore, define what is meant by a self-financing trading strat- egy.

(ii)   Show that a trading strategy (ϕ, ψ) is self-financing if and only if its wealth process X satisfies

dXt  = (rXt + (µ _ r) ϕt St ) dt + σϕt St dWt ,    t > 0. [5]

(b)   State the Black-Scholes partial differential equation (PDE). [2]

(c)   Consider a European option whose payoff is h (ST ) at time T. In the following assume that its value C (t, x) at time t < T is a C1,2 function of t and x. Suppose that the processes ϕ, ψ are defined by

∂C (t, St ) + σ 2 St(2) (t, St )

Suppose also that X is the wealth process corresponding to the trading strategy  (ϕ, ψ) and  C  satisfies the Black-Scholes PDE with terminal condition

C (T, x) = h (x) .

Show that the wealth process X satisfies

C (t, St ) = Xt , t e [0, T]

and (ϕ, ψ)  is a self-financing strategy.

[4]

(d)   Define the risk neutral measure Q in the context of the Black-Scholes model.  Give  an  explicit  expression  for ,  i.e.   the  Radon-Nikodym derivative of the risk neutral measure with respect to the physical mea- sure (no proof required). [3]

(e)   State and prove the put-call parity theorem. [5]

(f)   (i)   State the theorem on the replicability of options in the Black-Scholes model (no proof required).

(ii)   An Asian put option is a put option with exercise time T > 0 and strike K > 0 on the average price of the stock from time 0 to time T. What is the payoff for this option? Prove that the Asian put option is replicable in the Black-Scholes model. You may use the theorem on the replicability of options provided you show its assumptions are satisfied.

[6]








SOLUTIONS: MAT00028M


(a)   A Brownian Motion is a stochastic process (Wt )to0 on a probability space (Ω , F, P) such that

1. W0  = 0 a.s.

2.  for all k  e N and all 0 = t0   < t1 < . . . < tk the r.v.s Wt1 _ Wt0 , Wt2 _ Wt1 , . . . , Wtk _ Wtk 1   are independent (independent increments)

3.Wt _ Ws  ~ N(0, t _ s)  for all  t > s > 0

4.Trajectories are continuous a.s.

We say that (Wt )to0  is a Brownian Motion with respect to the ltration (Ft )to0  (or (Ft )to0-Brownian motion) if:

1. the random variable Wt  is Ft-measurable for every t > 0,

2. the random variable Wt  _ Ws  is independent of Fs  for all 0c.


(b)   If ξ e Mst(2)ep  (0, o), then

we can nd N e N and a sequence of times

0 = t0  < t1 < . . . < tN

such that

ξ(t) =

Hence, we have on the one hand

E|I(ξ)| 2     =   E ξ(ti )(W (ti+1) _ W (ti ))!2

= E ξ(ti )2 (W (ti+1) _ W (ti ))2

+   2     z     E ξ(ti )ξ(tj )[W (ti+1) _ W (ti )][W (tj+1) _ W (tj 2])

Since ξ(ti )2  is Fti _measurable and W (ti+1) _ W (ti ) is independent of Fti   we have for i = 0, . . . , N _ 1

┐           ┌        ┐   ┌                            ┐

=   E ξ(ti )2 (ti+1 _ ti ).

For the terms in the second sum we note that the random variables

ξ(ti ), W (ti+1) and W (ti ) are Ftj _measurable and W (tj+1) _ W (tj ) is


SOLUTIONS: MAT00028M

independent of Ftj . Hence,

E ξ(ti )ξ(tj )[W (ti+1) _ W (ti )][W (tj+1) _ W (tj )]

Combining the two calculations we see that

E|I(ξ)| 2  = E ξ(ti )2 (ti+1 _ ti ).

On the other hand, note that

E |ξt | 2  =

and therefore

ì0 o E|ξt | 2 dt

=   E|ξt0 | 2 (t1 _ t0 ) + E|ξt1 | 2 (t2 _ t1 ) + . . . + E|ξtN 1 | 2 (tN  _ tN _1 ).

c

7 Marks

s             '

(c)   ξ is not a random step process as it is not adapted  (6W13   is not F8 measurable).  For η we check that W2  is F3  measurable and 2W4(2)  is F6 measurable, the constants are measurable anyway, hence η is adapted. Noting that E (W4(4)) < o and E(W2 )2 < o (moments of normal random variables) the integrability condition also holds. The step property holds for the partition 0 < 3 < 6 < 9 and the process vanishes for t > 9. Hence, η is  a random step process. Finally, E (I (η)) = 0 either by general prop-

erties of the stochastic integral or by direct calculation. Hence, using the

Ito Isometry Var(I (η)) = E I (η)2 = 16 × 3 + 2 × 3 + 4EW4(4) × 3 =

s            '

(d)   (i) Theorem 0.1 |上tao 上emmb in simple form女

与ssume thbt f (t, x) is in C1 2, (R )! i冬e冬  the derivbtives2 , ,

ezist bnd bre continuous冬 Ωe忐ne b process η 6y

η(t) = f (t, W (t)), t > 0.

7hen η(t)! t > 0 is bn 上tao process bnd

η(t) = f (0, 0) + ì0 t [ (s, W (s)) + (s, W (s))]ds            + ì0 t (s, W (s)) dW (s), t   >   0.

(ii)   We apply the the Ito lemma in its simple form with f (t, x) = x2 . Note that f e C1,2  and fx (t, x) = 2x, fxx (t, x) = 2, ft  = 0. Hence,

Wt2  = ì0 t 2Ws dWs + ì0 t dt

and Wt2  _ t can be written as a stochastic integral.  It is easy to check that the integrand 2Ws  is in Mlo(2)c (0, o) . Adaptedness and continuity is satisfied by definition of the BM. Moreover, E(4Ws )2  = 4s so the integrability condition holds as well. We deduce that the stochastic integral is a martingale.

(iii)   For the nal part we note that the process (W) given by (W)t  = t, t > 0 is an adapted, continuous and increasing process with (W) 0  = 0. Moreover, Wt2  _ (W)t  is a martingale by the previous part of the question.   Hence,  (W)  satisfies the defintion of the quadratic

variation of the square-integrable martingale W.

c

8 Marks

s             '

c

Total: 25 Marks

2.               (a)   We say that a process (ξt )to0  belongs to class M2 (0, o) if and only if

(i)    (ξt )to0  is adapted to the ltration (Ft )to0 .

(ii)   the trajectories of (ξt )to0  are left continuous or right continuous a.s. (iii)   E |0o |ξt | 2 dt < o.

We say that a process (ξt )to0  belongs to class Mlo(2)c (0, o) if and only if (i)    (ξt )to0  is adapted to the ltration (Ft )to0 .

(ii)   the trajectories of (ξt )to0  are left continuous or right continuous a.s. (iii)   For every T > 0, E |0T |ξt | 2 dt < o.

Note that the process A is a continuous function of Wt  and Wt  is BM (which is adapted and has by definition a.s.  continuous sample paths). Hence, we can deduce that At  is adapted with continuous sample paths. It remains to check the integrability condition.   For every T > 0 using that the cosine function is bounded above by one we have

ì0 T E sin2 exp(7(Wt )3 )、、dt s ì0 T dt = T < o.

Brownian motion is not in Mlo(2)c (0.o) as E |0o |Wt | 2 dt = |0o c(tdt) = o.

6 Marks

s             '

(b)   If f (x) = sin (x) we have = cos (x) , = _ sin (x) and = 0, so f e C1,2 . Hence, we can apply Ito’s formula and get

dXt  = _ sin (Wt ) dt + cos (Wt ) dWt .

Substituting Xt  for sin (Wt ) in the above equation and observing that

sin (W0 ) = 0 we see that Xt  solves the SDE as required.

c

4 Marks

s             '

(c)   Applying the Ito lemma in its general form with f (t, x) = x2  to the Ito processes (X + Y) and (X _ Y) yields

d (X + Y)2t     = (aX (t) + aY (t)) 2 (Xt + Yt )) + (bX (t) + bY (t))2dt + (bX (t) + bY (t)) 2 (Xt + Yt ) dWt

and

d (X _ Y)2t     = (aX (t) _ aY (t)) 2 (Xt _ Yt )) + (bX (t) _ bY (t))2dt + (bX (t) _ bY (t)) 2 (Xt _ Yt ) dWt

Subtracting the second from the rst equation and dividing by four we see that

d (XY)t     = [4aX (t) Yt + 4aY (t) Xt + 4bX (t) bY (t)] dt + (4bX (t) Yt + 4bY (t) Xt ) dWr

=   aX (t) Yt dt + bX (t) Yt dWt + aY (t) Xt dt + bY (t) Xt dWt

+bX (t) bY (t) dt

=   Yt dXt + Xt dYt + bX (t) bY (t) dt.

c

6 Marks

s             '

(d)   Note that the function

f (t, x) = exp _βx + β 2t

is in C1,2 . Hence, we can apply the Itˆo lemma to Gt   = f (t, Wt ) and deduce that

dGt     = β 2 Gt dt + β 2 Gt dt _ βGt dWt

=   β 2 Gt dt _ βGt dWt .

The stochastic  integration by parts formula for two processes  dξt   = a1 (t) dt + b1 (t) dWt  and dηt  = a2 (t) dt + b2 (t) dWt  states that

d (ξt ηt ) = ξt dηt + ηt dξt + b1 (t) b2 (t) dt.

Applying stochastic integration by parts to Zt  = Xt Gt  we have

dZt     =   Xt dGt + Gt dXt _ β 2 GtXt dt

=   Xt β 2 Gt dt _ Xt βGt dWt + Gt (Xt )γ dt + Gt βXt dWt _ β 2 GtXt dt = β 2 Zt dt _ βZt dWt + Gt (Xt )γ dt + βZt dWt _ β 2 Zt dt

=   Gt (Xt )γ dt

=   (Gt )1_γ (Zt )γ dt.

We now solve this ODE for 0 < γ < 1 by considering

dZt  = Zt(γ)Gt(1) _γ dt,    Z0  = 0.

Using separation of variables

ì0 t = ì0 t Ft1_γ dt

and hence,

Z x1_γ  = (1 _ γ) ì0 t Ft1_γ dt.

Therefore,

1

Zt  = x1_γ + (1 _ γ) ì0 t Ft1_γ dt1 γ .

Finally, if γ = 1 we know from the examples in the lectures that Xt  is the stochastic exponential

Xt  = x0 exp ╱╱1 _ β 2 /2t + βWt.

c

9 Marks

s             '

c

Total: 25 Marks

3.               (a)   (i)   The wealth process X(t), t > 0 corresponding to the trading strategy (ϕ, ψ), is defined by

X(t) := ψ(t)B(t) + ϕ(t)S(t),    t > 0.                   (3)

A trading  strategy  (ϕ, ψ)  is self-financing if the  corresponding wealth process satisfies

dXt  = ψ(t) dBt + ϕ(t) dSt ,    t > 0                      (4)

or equivalently

ψt Bt + ϕt St _ (ψs Bs + ϕs Ss ) =      ϕu dSu +      ψu dBu

s                            s

for s, t > 0.

(ii)   The claim follows from the equality

(rXt + (µ _ r) ϕt St ) dt + σϕt St dWt = (r (ϕt St + ψt Bt ) + (µ _ r) ϕt St ) dt + σϕt St dWt

= (rψt Bt + µϕt St ) dt + σϕt St dWt = ψtrBt dt + ϕt (µSt dt + σSt dWt ) = ψt dBt + ϕt dSt.

c

5 Marks

s             '

(b)   The Black-Scholes PDE is given by

+ rx + σ 2 x2 _ rC = 0,    t e [0, T] , x > 0.        (5)

c

2 Marks

s             '

(c)   The wealth process corresponding to (ϕ, ψ) satisfies

C (t, St ) + σ 2 St(2) (t, St )

x                                      rBt

where we have used that C satisfies the Black-Scholes PDE in the last step. To show that the self-financing property holds we note that using the Ito formula

ì0 t ϕs dSs + ì0 t ψs dBs     =   ì0 t (t, Ss ) dSt

+ ì0 t (s, Ss ) + σ S2s(2) (t, Ss )ds

=   C (t, St ) _ C (0, S0 ) .

c

4 Marks

s             '

(d)   The risk neutral measure Q in the B-S model is the unique P equivalent measure under which the stock price discounted by the risk free interest

rate is a martingale.  In this case we have (restricting the measures to FT )

= exp _ WT  _ T\

c

3 Marks

s             '

(e)   Put-Call parity theorem:

Let T > 0 and let C(t), t e [0, T], be the value of a call option with strike price K and maturity time T. Let P (t) be the value of a put option with the same strike and maturity. We denote by S(t) the price of the share at time t, and r the risk-free interest rate. Then

C(t) _ P (t) = S(t) _ Ker(t_T) .

(6)


Proof: Suppose that at time t = 0 we buy a call option and sell a put option with the same maturity time T and strike price K .  At time T, the payoff for this portfolio is S(T) _ K, as

C(T) _ P (T) = |S(T) _ K|+ _ |K _ S(T)|+  = S(T) _ K.

Also  at time  0,  we  can  can buy  a  unit  of share  and borrow  M  :=

Ke_rT  using bonds (hence, at time T we have to return exactly erT M =

erT Ke_rT  = K).  The payoff for this second portfolio  at time T is also

S(T) _ K .  As we assume the no br6itrbge principle we conclude that

the prices of these two portfolios have to be the same for every t e [0, T],

s            '

(f)   (i) Theorem 0.2 Under the bssumptions of the 去lbck」scholes model

suppose thbt h is bn option with ezpiry time T > 0 suppose more」 over thbt

EQT |h|2  < o,

bnd h is b non」negbtive rbndom vbrib6le b冬s冬冬  7hen the option h is replicb6le冬 koreover! if

Xt  = ϕt St + ψt Bt , t e [0, T]

is the weblth process corresponding to the bdmissi6le replicbtion strbt」 egy (ϕt , ψt ) |in pbrticulbr sbtisfying XT  = h)! then

Xt  = EQT e_r(T _t)h Ft,    t e [0, T].                   (7)